This page shows a recording of a live class. We're working hard to create our standard, concise explanation videos for the questions in this PrepTest. Thank you for your patience!

This is a flaw question, and we know this because of the question stem: which one of the following indicates an error in the reasoning leading to the prediction above?

We’re told that the national savings rates for certain countries have dropped. Since older people have fewer reasons to save than do younger people, this trend of decreasing savings rate will continue if the average age of the population continues to get older.

If older people have fewer reasons to save, does that mean that what their saving up for is worth less? Maybe older people just have distinct priorities, for example retirement, house, and car. All of these are huge investments. Younger people might be saving up for more things, but these things could be worth a lot less. It is not logical to assume that quantity is proportional to value.

Answer Choice (A) is descriptively accurate; however, it is not a flaw. The reason the argument is flawed has nothing to do with listing out many reasons younger people have and the strongest of those reasons. It’s about how these reasons compare to older people’s reasons.

Answer Choice (B) is not descriptively accurate. Nowhere does the argument assume that a negative savings rate can’t happen.

Answer Choice (C) is descriptively accurate but it isn’t the flaw. The average age rising comes into play in the conclusion where this is the sufficient condition: if the age rises, then the savings rate will drop lower. The author is not outright claiming that age is rising and does not need to do this.

Correct Answer Choice (D) addresses the argument overlooking the difference between amount and value. This is perfect.

Correct Answer (E) is descriptively accurate but it’s not the flaw. The different kind of taxes being compared does have anything to do with age increasing.


1 comment

This page shows a recording of a live class. We're working hard to create our standard, concise explanation videos for the questions in this PrepTest. Thank you for your patience!

The flaw/descriptively weakening questions, we know this because of the question stem: A flaw in the argument is that the author...

We’re first told that a common procedure for figuring out whether a food additive should be banned is to compare health-related benefits with risks. We’re then given a specific food additive: Yellow Dye No. 5. This could cause allergic reactions for some people. However, the yellow die enhances the drinking experience for some consumers. The argument concludes that because the benefits of enjoying the drink greatly outweighs the allergic reaction risk, the food dye should not be banned.

This argument is internal incoherent. Remember the first sentence we’re giving says that in order to assess whether a food additive should be banned, we should compare it’s health-related benefits to the risks. Enjoyment is not really a “health-related” benefit, it’s just a benefit. Based on what criteria we have in the argument, the conclusion does not logically follow.

Answer Choice (A) is not right because the argument does not imply this; this answer choice is not descriptively accurate. The stimulus implies that enjoyment is a health-related benefit, which isn’t reasonable.

Correct Answer Choice (B) demonstrates the flaw within the argument - the equivocation of enjoyment of a beverage to a health-related benefit.

Answer Choice (C) is descriptively accurate: the stimulus does ignore the possibility that some additives harm people. But this is not a flaw in the argument; this is not even within the scope of what the argument is talking about.

Answer Choice (D) is not good either; remember, we have to accept our premises. The claim about yellow dye has to be accepted.

Answer Choice (E) is debatable on whether it’s relatable. Let’s be generous and say it is. This still isn’t a flaw. What about the some that do not pay attention to the warning labels and are allergic to the dye? Would the enjoyment benefit still outweigh the risks?


Comment on this

This page shows a recording of a live class. We're working hard to create our standard, concise explanation videos for the questions in this PrepTest. Thank you for your patience!

This is a weakening question, since the question stem demands: Which one of the following, if true, most seriously undermines the conclusion that change in the direction of the Earth’s magnetic field happened very slowly?

The stimulus begins with the conditional indicator when, and tells us that whenever lava solidifies, it necessarily magnetizes in the direction that the earth’s magnetic field points at the moment of solidification. Interesting! The next sentence gives us some more information about this event, namely that lava flows from different volcanos that erupted at different points over the past several million years are magnetized in different directions. From this evidence, the next sentence concludes that the direction’s of the earth’s magnetic field has changed over time. Seems like a fairly reasonable conclusion! The next sentence begins with the support indicator since, and whenever we see a sentence that takes the form “since X, Y”, we should expect X to be the final premise introduced before the conclusion Y. That’s exactly what we get here. The final premise tells us that lava flows that are separated by only a few thousand years have very similar magnetization directions. Based on this and our sub-conclusion from the last sentence, the stimulus ends with the main conclusion that the direction of the Earth’s magnetic field changes gradually. Our job is to select the answer choice which most weakens this conclusion; we want something that suggests non-gradual change. Let’s anchor ourselves in the argument, and consider the implications of the answer choices:

Answer Choice (A) This answer might be tempting because the movement of the liquids is described as chaotic, but there’s no reason why a gradual change couldn’t have a chaotic cause.

Answer Choice (B) If anything this answer would suggest gradual change, since we have not detected any real change during the short period we have been monitoring the field.

Answer Choice (C) The problem with this answer is that we don’t know what a complete reversal means in this case; maybe it was a very small change. And even if it was a significant change, it occurred over a long time period (a few million years), so it could still be entirely consistent with gradual change.

Correct Answer Choice (D) This answer does what C fails to; it introduces a significant change in a short timespan. A significant change in the span of weeks is completely inconsistent with a theory of gradual change over thousands of years.

Answer Choice (E) This answer is just entirely irrelevant to the gradual change theory we want to undermine. Who cares how long some lava flows will take to solidify, we want to know whether the magnetic field the point to is changing rapidly!


1 comment

This page shows a recording of a live class. We're working hard to create our standard, concise explanation videos for the questions in this PrepTest. Thank you for your patience!

This is a strengthening question, as we are asked: Which one of the following principles, if accepted, would contribute most to Shanna’s defense of her position against that of Jorge?

This stimulus is a dialogue, and our job is to specifically strengthen Shanna’s position against Jorge’s argument. Shanna argues that just by owning a piece of art, you have the right to destroy it should you want to. If we think about it, I’m sure many of us would disagree with Shanna. But our job is to support this position; if it’s yours, you can destroy it.

Jorge responds by arguing that although ownership gives you the right to possess an unique piece of art, you don’t have the right to destroy it. This is because a unique piece of art belongs to posterity (basically, humanity as a whole including those who don’t even exist yet), and so it must be preserved regardless of what you want. Ok, so Jorge thinks the value of unique art to humanity as a whole outweighs the individual rights of the owner. We should look for a principle that empowers the owner. Let’s see what we get:

Answer Choice (A) Well according to Shanna they would still be in their rights to destroy a great piece of art if they owned it and caring for it was inconvenient! But it should be obvious this doesn’t help our position at all.

Answer Choice (B) This principle supports Jorge!

Answer Choice (C) This is arguing that we shouldn’t destroy art because of its value to humanity - that’s Jorge’s position and not Shanna’s!

Answer Choice (D) This is undermining the rights of legal owners, which is the opposite of what we want to do.

Correct Answer Choice (E) The qualification about health and safety might throw you off but this is correct. If individuals can do what they want with their property so long as it doesn’t physically harm others, then they are allowed to destroy artwork they own.


1 comment

This page shows a recording of a live class. We're working hard to create our standard, concise explanation videos for the questions in this PrepTest. Thank you for your patience!

This is a weakening question, since the stem says: Which one of the following, if true, most seriously weakens the argument?

The first thing we learn is that the fines for environmentally damaging accidents are so high that it is cheaper to prevent an accident than it is to deal with the results. The second sentence begins with the conclusion indicator therefore, and concludes that businesses will now invest in preventing accidents. This sentence has a little clause in between commas with the support indicator since, which introduces the final piece of support which is that businesses care about money. It makes intuitive sense, if accidents are going to lose you a lot of money in fines, more than preventing them will, then we would expect those that care about money to prefer prevention to fines. Our job is to weaken this prediction about what businesses are going to do. Let’s see the answers:

Correct Answer Choice (A) Bingo. The argument depends on assuming that businesses will choose to invest in prevention rather than just hoping there won’t be an accident. If they significantly underestimate how likely accidents are, well then they are likely going to assume they don’t have to worry about accidents and the fines, and therefore won’t invest in prevention.

Answer Choice (B) This strengthens the argument, since the prevention of future accidents is a long-term strategy.

Answer Choice (C) This is totally compatible with the argument, the whole point is that businesses are going to invest in prevention because it makes business sense.

Answer Choice (D) This might be appealing if you infer that considering something an ordinary business expense means you don’t mind paying it; the problem is that we’ve been told that businesses care about their profits, and paying fines is more expensive than preventing them. So even if they consider fines an ordinary expense, the argument can still conclude they will choose to pay the cost of prevention instead.

Answer Choice (E) This does nothing to change the fact that prevention is cheaper than fines, and businesses want to maximize their overall profits.


1 comment

This page shows a recording of a live class. We're working hard to create our standard, concise explanation videos for the questions in this PrepTest. Thank you for your patience!

This is a weakening question, indicated in the question stem by: Which one of the following, if accepted by Clay Moltz, would require him to reconsider his conclusion. Bit of a weird question stem, but we should recognize that if introducing something forces him to reconsider his conclusion, then it is weakening his argument.

Our stimulus begins with a bunch of context about a mathematical model which tries to predict the chances of extraterrestrial life existing. To be honest, the model seems to kind of suck. It relies on some pretty large assumptions about how life could exist and whether other star systems resemble ours. It is important for us to remember that we are here to weaken Clay Moltz’ reasoning, not the model’s. It seems Moltz also thinks this model sucks, but his reasoning is itself pretty poor. He infers that because we have not detected any planets outside the solar system, there must not be any life out there. It is again important to recognize that Moltz specifically restricts his conclusion to life as we know it. If some really weird stuff exists out there, that doesn’t contradict his conclusion unless it is life similar to that of Earth. Moltz argument is a classic case of confusing an absence of evidence for evidence of an absence. We can’t infer that just because we haven’t detected any planets, that we know whether there are any out there. Let’s see how the correct answer undermines this argument:

Answer Choice (A) Remember that Moltz only concludes about life as we know it; this answer choice would not force him to reconsider this conclusion.

Answer Choice (B) This could be true and Moltz’ argument would be fine; in fact, he seems to deny that there are planets out there, and therefore this answer is what we would expect to be true if his conclusion is.

Correct Answer Choice (C) This gives an alternative explanation for our not having detected any planets that is much more appealing than Moltz’s ‘there are none to detect’ hypothesis. Of course, we wouldn’t have detected any if we can’t detect any!

Answer Choice (D) This answer might be appealing if you forgot what it is we are supposed to weaken. Our target is Moltz, not the model and its large assumptions.

Answer Choice (E) Alright, but that wouldn’t explain why we haven’t detected any planets. Moltz could accept this without it posing any challenge to his support or conclusion.


4 comments

This page shows a recording of a live class. We're working hard to create our standard, concise explanation videos for the questions in this PrepTest. Thank you for your patience!

This is a necessary assumption question. We see this because the argument in the stimulus depends on the assumption expressed by the correct answer choice. The assumption must be true for the argument to hold. And if the answer is not true, the conclusion will not be able to follow.

This sounds like a cool technology. The scanner can recognize patterns. Makes sense. See it once; store the data; see it again; match the new scan to the former. Simple enough. No two eyes are the same, hence its reliability because it can’t be confused by any possibility of seeing the same pattern in different eyes. The conclusion in the final sentence may initially seem reasonable enough. Isn’t this exactly what these scanners do? If you don’t see a gap here, that’s okay.

Our approach for NA questions is to keep an open mind going into the answer choices. This is particularly effective in situations like this where we may not see a gap. There is potentially a few things we might anticipate here, but we do not want to enter the AC’s searching for anything in particular. Let’s consider the Answers and let them speak to us. The correct answer choice will suggest the solution. If we’re open to it, we only need to be able to utilize the prompt to follow through and recognize the solution.

Correct Answer Choice (A) This is the right answer. To use negation: If the blood vessels could change to such an extent that the scanner can no longer match the pattern of the current scan to the previous one, then a retina scanner cannot necessarily “be used successfully to determine for any person whether it has ever scanned a retina of that person before.” If I get the right kind of eye disease, they won’t recognize me and I’m back off the grid. This would create a subset of exceptions which would not fit. Because the conclusion is universal, there are no exceptions allowed. This answer tells us that such an exception can not occur, which is necessary for the conclusion to follow.

Answer Choice (B) This is tempting because we understand that most people have two eyes and we recognize a distinction between recognizing eyes and recognizing people. If this led you astray, check the stimulus again. We’re only concluding that the retina scanner can be used to tell if it has scanned a retina of a person before. We don’t limit its methodology in how it might determine this: It could simply scan both eyes, no problem.

An even simpler reason to eliminate this answer choice, however, is that the stimulus has already told us, “no two eyes have identical patterns.” So actually, we already know that every person has different patterns of blood vessels in each eye. Those are two eyes, after all, and “no two eyes have identical patterns.” So not only is this not necessary, it contradicts the stimulus.

Answer Choice (C) We don’t care. Our conclusion is not indicating we can create a global database of scans so that we can ID anyone in the world. It only needs to be able to recognize someone it’s already scanned before. If someone isn’t already in the system, it doesn’t need to be able to recognize them per our conclusion.

Answer Choice (D) What if they’re not? Then some people would not only have different patterns of vessels but also different numbers of vessels. Would that mean the scanners couldn’t recognize people? If anything, that would give them more variation to go on. So if this isn’t true, it may even be helpful. If the negated form of this answer strengthens the argument, then this does not have to be true.

Answer Choice (E) I don’t know how this matters at all. What if there is? How does that bear on whether or not a scanner could recognize you with a second look? If you’ve been scanned by two scanners, then two scanners should be able to recognize you. No big deal. So this is not necessary.


1 comment

This is a resolve reconcile explain question, indicated by the stem asking: Which one of the following, if true, most helps to reconcile the restorer’s decision with goal stated in the passage?

The stimulus tells us that the great Renaissance painter Michelangelo’s paintings in the Sistine Chapel are being restored. Awesome, who doesn’t love Michelangelo! The purpose of the restoration is to uncover the Sistine Chapel as Michelangelo originally painted it, and part of this process is removing later artists’ additions. Strangely however, the restorers have decided to leave one addition alone: that of da Volterra. We’re looking for an answer that will explain the restorers’ decision in a way that is compatible with their overall purpose of uncovering Michelangelo’s work.

Correct Answer Choice (A) Well if he stripped away all the paint then there is no Michelangelo below his addition, and therefore no benefit of removing his addition for the restorers. Their decision makes sense now!

Answer Choice (B) Cool fact but this doesn’t give us any reasons why the restorers wouldn’t remove his addition.

Answer Choice (C) I guess it might fit in more but the purpose of the restorers was to uncover Michelangelo’s original work; whether additions are stylistically similar shouldn’t affect whether they are kept in.

Answer Choice (D) The relative ranking of the different subsets of Michelangelo and Volterra’s art would have no impact on the restorers’ project. They want to uncover originals and seem to have no problem removing additions, except for Volterra’s.

Answer Choice (E) And yet the stimulus tells us they removed those other artists. That’s exactly the kind of discrepancy we want an explanation for!


1 comment

This page shows a recording of a live class. We're working hard to create our standard, concise explanation videos for the questions in this PrepTest. Thank you for your patience!

This is a strengthening question, as the stem asks us: Which one of the following, if true, supports the conclusion in the passage?

This is one of those questions where you can’t help but let out a sigh when you turn the page on your LSAT and it greets you. A long stimulus doesn’t necessarily mean a difficult question, but it usually means a time consuming one. We should always focus on sifting through the unnecessary details, and try to get a good grasp on the actual argument. This stimulus begins with a lot of context about computer software and how its standardization can make it vulnerable to viruses. Because computer software has become standardized, such that a business might have all its computers running on the same or similar software, a virus which infects one computer will have a high likelihood of being able to spread throughout the network to the other similar computers, giving a vandal the potential to destroy data on all the computers. Seems like a big problem! Luckily, our author has a solution; just make the software differ slightly between computers. By introducing minor variations that are compatible with business, the possibility of a virus destroying all data can be eliminate. This solution also wouldn’t entail any loss in computer compatibility. Our author concludes that this minor variation should be adopted. Our job is to strengthen this conclusion. Let’s see what the answer choices have in store for us:

Answer Choice (A) Interesting, but we’ve been explicitly told introducing minor variations won’t negatively impact this compatibility.

Correct Answer Choice (B) This strengthens our argument, since if true it would mean that whatever the costs associated with introducing minor variation are, they are worth avoiding the damage.

Answer Choice (C) Again, the author’s solution will not impact compatibility.

Answer Choice (D) Our conclusion is that a certain strategy will prevent a particular problem. The potential existence of other problems does nothing to strengthen the proposal.

Answer Choice (E) Interesting, but I don’t see how this would strengthen our proposed solution.


1 comment